Đến nội dung

Hình ảnh

[TOPIC] ÔN THI BẤT ĐẲNG THỨC $\boxed{\text{THPT CHUYÊN}}$ LỚP $10$ năm $2018-2019$

bất đẳng thức holder cosi bunhiacopxki

  • Chủ đề bị khóa Chủ đề bị khóa
Chủ đề này có 318 trả lời

#121
DOTOANNANG

DOTOANNANG

    Đại úy

  • ĐHV Toán Cao cấp
  • 1609 Bài viết

Bài 54. Cho các số dương $a\,,\,b\,,\,c$. Chứng minh rằng: $\frac{a}{b}\,+ \,\frac{b}{c}\,+\, \frac{c}{a}\,+\, 1\,\geqq \,\frac{2\sqrt{2}}{3}\,(\sqrt{\frac{b+c}{a}}\,+\, \sqrt{\frac{c+a}{b}}\,+\, \sqrt{\frac{a+ b}{c}})$


Bài viết đã được chỉnh sửa nội dung bởi MoMo123: 21-04-2018 - 20:47


#122
hoangkimca2k2

hoangkimca2k2

    Sĩ quan

  • Thành viên
  • 477 Bài viết

Bài 55: Cho $a,b,c$ là các số thực dương thỏa mãn $a+b+c=3$. Chứng minh rằng $\sqrt[5]{a(a+c)(2a+b)}+\sqrt[5]{b(b+a)(2b+c)}+\sqrt[5]{c(c+b)(2c+a)}\leq 3\sqrt[5]{6}$


Bài viết đã được chỉnh sửa nội dung bởi MoMo123: 20-04-2018 - 20:11

  N.D.P 

#123
hoangkimca2k2

hoangkimca2k2

    Sĩ quan

  • Thành viên
  • 477 Bài viết

Bài 57: Cho $a,b,c>0$ và $ab+bc+ac=abc$. Chứng minh rằng $P=\frac{a^{4}+b^{4}}{ab(a^{3}+b^{3})}+\frac{b^{4}+c^{4}}{bc(c^{3}+b^{3})}+\frac{c^{4}+a^{4}}{ac(a^{3}+a^{3})}\geq 1$


  N.D.P 

#124
doraemon123

doraemon123

    Trung sĩ

  • Thành viên
  • 169 Bài viết

Bài 38:
attachicon.gifIMG_20180418_224653.jpg
P/s: Xin lỗi m.n mình đang dùng đt nên không gõ Latex được

Mình xin đưa ra lời giải bài trên:

Áp dụng BĐT Holder ta có:

$(1+3)(1+3)(1+3)(a^4+3)\geq (a+3)^4\Rightarrow \sqrt[4]{a^4+3}\geq \frac{a+3}{\sqrt[4]{64}}$

Cmtt: $\sqrt[4]{b^4+3}\geq \frac{b+3}{\sqrt[4]{64}}; \sqrt[4]{c^4+3}\geq \frac{c+3}{\sqrt[4]{64}}$

Cộng vế theo vế ta được: $\sqrt[4]{a^4+3}+\sqrt[4]{b^4+3}+\sqrt[4]{c^4+3}\geq \frac{(a+b+c)+3+3+3}{\sqrt[4]{64}}$

Áp dụng BĐT AM-GM, có: $(a+b+c)+3+3+3\geq 4\sqrt[4]{27(a+b+c)}\Rightarrow \sum \sqrt[4]{a^4+3}\geq \frac{4\sqrt[4]{27(a+b+c)}}{\sqrt[4]{64}}=\sqrt[4]{108(a+b+c)}$


Bài viết đã được chỉnh sửa nội dung bởi doraemon123: 20-04-2018 - 14:14

$\sqrt{MF}$  math is like reality that so many problem to solve $\sqrt{MATH}$

                                               (~~) (~~) :ukliam2: :ukliam2: :ukliam2: :ukliam2:  (~~) (~~) 


#125
tr2512

tr2512

    Thượng sĩ

  • Thành viên
  • 272 Bài viết

Bài 58

Cho các số thực không âm a, b, c thỏa mãn $a+b+c=3$. Chứng minh:

$\frac{a}{{\sqrt {2a + b} }} + \frac{b}{{\sqrt {2b + c} }} + \frac{c}{{\sqrt {2c + a} }} \le \sqrt 3$


Bài viết đã được chỉnh sửa nội dung bởi MoMo123: 20-04-2018 - 20:11


#126
doraemon123

doraemon123

    Trung sĩ

  • Thành viên
  • 169 Bài viết

Bài 57: Cho $a,b,c>0$ và $ab+bc+ac=abc$. Chứng minh rằng $P=\frac{a^{4}+b^{4}}{ab(a^{3}+b^{3})}+\frac{b^{4}+c^{4}}{bc(c^{3}+b^{3})}+\frac{c^{4}+a^{4}}{ac(c^{3}+a^{3})}\geq 1$

Ta có:$ab+bc+ac= abc\Rightarrow \frac{1}{a}+\frac{1}{b}+\frac{1}{c}=1$

Ta có: $2(a^4+b^4)\geq (a^3+b^3)(a+b)\Rightarrow \frac{a^4+b^4}{a^3+b^3}\geq \frac{a+b}{2}\Rightarrow \frac{a^4+b^4}{ab(a^3+b^3)}\geq \frac{a+b}{2ab}$

CMTT: $\frac{b^4+c^4}{bc(b^3+c^3)}\geq \frac{b+c}{2bc}; \frac{a^4+c^4}{ac(a^3+c^3)}\geq \frac{a+c}{2ac}$

Cộng vế theo vế ta được: $\sum \frac{a^4+b^4}{ab(a^3+b^3)}\geq \frac{1}{2}(\sum \frac{a+b}{ab})\doteq \frac{1}{2}(2.\sum \frac{1}{a})=1$

Dấu ''='' xảy ra khi a=b=c=3


Bài viết đã được chỉnh sửa nội dung bởi doraemon123: 20-04-2018 - 14:16

$\sqrt{MF}$  math is like reality that so many problem to solve $\sqrt{MATH}$

                                               (~~) (~~) :ukliam2: :ukliam2: :ukliam2: :ukliam2:  (~~) (~~) 


#127
tr2512

tr2512

    Thượng sĩ

  • Thành viên
  • 272 Bài viết

Bài 42:

Cho các số thực không âm $a, b, c$ thỏa mãn $a+b+c=3$. Chứng minh:

$\frac{{a + b}}{{\sqrt {2{\rm{a}} + b} + \sqrt {2b + a} }} + \frac{{b + c}}{{\sqrt {2b + c} + \sqrt {2c + b} }} + \frac{{a + c}}{{\sqrt {2{\rm{a}} + c} + \sqrt {2c + a} }} \le \sqrt 3$ (Lê Ngọc Đức)

Lời giải cho bài toán:

$\frac{{b + c}}{{\sqrt {2b + c} + \sqrt {2c + b} }} + \frac{{a + c}}{{\sqrt {2a + c} + \sqrt {2c + a} }} \le \sqrt 3 \\ \Leftrightarrow \frac{{a + b}}{{\sqrt {3\left( {2a + b} \right)} + \sqrt {3\left( {2b + a} \right)} }} + \frac{{b + c}}{{\sqrt {3\left( {2b + c} \right)} + \sqrt {3\left( {2c + b} \right)} }} + \frac{{c + a}}{{\sqrt {3\left( {2a + c} \right)} + \sqrt {3\left( {2c + a} \right)} }} \le 1\\ \Leftrightarrow \sum \frac{{a + b}}{{\sqrt {\left( {a + b + c} \right)\left( {a + b + a} \right)} + \sqrt {\left( {a + b + c} \right)\left( {a + b + a} \right)} }} \le 1$

Áp dụng bất đẳng thức C-S:

$\sqrt {\left( {a + b + c} \right)\left( {a + b + a} \right)} \ge a + \sqrt {\left( {b + c} \right)\left( {b + a} \right)} \\ \sqrt {\left( {b + a + c} \right)\left( {b + b + a} \right)} \ge b + \sqrt {\left( {a + c} \right)\left( {a + b} \right)}$

$\Rightarrow \sum \frac{{a + b}}{{\sqrt {\left( {a + b + c} \right)\left( {a + b + a} \right)} + \sqrt {\left( {a + b + c} \right)\left( {a + b + a} \right)} }} \le \sum \frac{{a + b}}{{a + b + \sqrt {\left( {b + c} \right)\left( {a + b} \right)} + \sqrt {\left( {a + c} \right)\left( {a + b} \right)} }} = \sum \frac{{\sqrt {a + b} }}{{\sqrt {a + b} + \sqrt {b + c} + \sqrt {c + a} }} = 1$

Hoàn tất chứng minh.



#128
Khoa Linh

Khoa Linh

    Thiếu úy

  • Thành viên
  • 601 Bài viết

Bài 58

Cho các số thực không âm a, b, c thỏa mãn $a+b+c=3$. Chứng minh:

$\frac{a}{{\sqrt {2a + b} }} + \frac{b}{{\sqrt {2b + c} }} + \frac{c}{{\sqrt {2c + a} }} \le \sqrt 3$

Ta có BĐT tương đương: $\sum \frac{a}{\sqrt{3(2a+b)}}\leq 1$

Mà ta có: $\frac{a}{\sqrt {3(2a + b)}}=\sqrt{\frac{a^2}{3(2a+b)}}\leq \frac{1}{2}\left (\frac{a}{2a+b}+\frac{a}{3} \right )$

Nên BĐT quy về chứng minh: $\sum \frac{a}{2a+b}\leq 1\Leftrightarrow \sum \frac{b}{2a+b}\geq 1$ 

Bất đẳng thức trên đúng theo Cauchy - Schwarz: $\sum \frac{b}{2a+b}=\sum \frac{b^2}{2ab+b^2}\geq \frac{(a+b+c)^2}{(a+b+c)^2}=1$


$\sqrt[LOVE]{MATH}$

"If I feel unhappy, I do mathematics to become happy. If I am happy, I

 

do mathematics to keep happy" - Alfréd nyi 


#129
HelpMeImDying

HelpMeImDying

    Trung sĩ

  • Thành viên
  • 108 Bài viết

 

$\boxed{\text{Bài 47}}$ : 

Giả sử $a,b,c$ là các số thực dương thỏa mãn đẳng thức $ab+bc+ca=1$. Chứng minh rằng 

                                                                  $2abc(a+b+c) \leq \frac{5}{9}+\sum a^4b^2$

 

Có: $a^{4}b^{2}+\frac{1}{3}abc^{2}+\frac{1}{9}ac\geq 3\sqrt[3]{\frac{a^{6}b^{3}c^{3}}{27}}= a^{2}bc$

Lập các bđt tương tự rồi cộng lại: $\sum a^{4}b^{2}+\frac{1}{9}(ab+bc+ca)\geq \frac{2}{3}abc(a+b+c)\Rightarrow \sum a^{4}b^{2}+\frac{5}{9}\geq \frac{2}{3}abc(a+b+c)+\frac{4}{9}$

Ta chứng minh: $\frac{2}{3}abc(a+b+c)+\frac{4}{9}\geq 2abc(a+b+c)\Leftrightarrow 1\geq 3abc(a+b+c)$ 

Đúng do: $1=(ab+bc+ca)^{2}\geq 3abc(a+b+c)$



#130
thanhdatqv2003

thanhdatqv2003

    Trung sĩ

  • Thành viên
  • 159 Bài viết

mk xin góp 1 bài 

Bài 59 : Cho a,b,c >0. Tìm GTNN của biểu thức

  P$\frac{a^3}{a^2+b^2}+\frac{b^3}{b^2+c^2}+\frac{c^3}{c^2+b^2}-\sqrt{a+b+c}$


Bài viết đã được chỉnh sửa nội dung bởi thanhdatqv2003: 21-04-2018 - 20:14

:ohmy: [Không tồn tại các nghiệm nguyên khác không x, y, và z thoả mãn xn + yn = zn trong đó n là một số nguyên lớn hơn 2.  (FERMAT)  :ohmy: 

 

 

 

 


#131
hoangkimca2k2

hoangkimca2k2

    Sĩ quan

  • Thành viên
  • 477 Bài viết

Bài 60: Cho $a,b,c>0$ và $a+b+c=3$. Chứng minh rằng: $\frac{a}{b+c}+\frac{b}{c+a}+\frac{c}{a+b}\geq \frac{21}{16}+\frac{27(a^{3}+b^{3}+c^{3})}{16(a+b+c)^{3}}$


Bài viết đã được chỉnh sửa nội dung bởi hoangkimca2k2: 21-04-2018 - 15:17

  N.D.P 

#132
hoangkimca2k2

hoangkimca2k2

    Sĩ quan

  • Thành viên
  • 477 Bài viết

mk xin góp 1 bài 

Bài 59 : Cho a,b,c >0. Tìm GTNN của biểu thức

  P$\frac{a^3}{a^2+b^2}+\frac{b^3}{b^2+c^2}+\frac{c^3}{c^2+b^2}-\sqrt{a+b+c}$

:D

Xét $A=\frac{a^3}{a^2+b^2}+\frac{b^3}{b^2+c^2}+\frac{c^3}{c^2+b^2}$. Ta có BDT quen thuộc $A\geq \frac{a+b+c}{2}$.

Chứng minh: Bất đẳng thức cần chứng minh tương đương:

$\frac{a^{3}}{a^{2}+b^{2}}-\frac{a}{2}+\frac{b^3}{b^2+c^2}-\frac{b}{2}+\frac{c^3}{c^2+a^2}-\frac{c}{2}\geq 0$

$\Leftrightarrow \frac{a^{3}-ab^{2}}{a^{2}+b^{2}}+\frac{b^3-bc^{2}}{b^2+c^2}+\frac{c^3-ca^{2}}{c^2+a^2}\geq 0$

$\Leftrightarrow (a-b)\frac{a^{2}+ab}{a^{2}+b^{2}}+(b-c)\frac{b^{2}+bc}{a^{2}+c^{2}}+(c-a)\frac{c^{2}+ac}{a^{2}+c^{2}}\geq 0$

$\Leftrightarrow \sum \left ((a-b)\frac{a^{2}+ab}{a^{2}+b^{2}}-(a-b)  \right )=\sum (a-b)^{2}\frac{b}{a^{2}+b^{2}}$  

Đến đây dùng $SOS$. Ta có được điều phải chứng minh.

Nên $P\geq \frac{a+b+c}{2}-\sqrt{a+b+c}$. Đến đây thì dễ rồi


  N.D.P 

#133
Tea Coffee

Tea Coffee

    Trung úy

  • Điều hành viên THPT
  • 772 Bài viết

Bài 55: Cho $a,b,c$ là các số thực dương thỏa mãn $a+b+c=3$. Chứng minh rằng $\sqrt[5]{a(a+c)(2a+b)}+\sqrt[5]{b(b+a)(2b+c)}+\sqrt[5]{c(c+b)(2c+a)}\leq 3\sqrt[5]{6}$

 $P$ = $\sqrt[5]{a(a+c)(2a+b)}+\sqrt[5]{b(b+a)(2b+c)}+\sqrt[5]{c(c+b)(2c+a)}$

$=>\sqrt[5]{36^{2}}.P=\sum \sqrt[5]{6.6.6a.(3a+3c)(4a+2b)}\leq \sum \frac{13a+3c+2b+12}{5}=18=>P\leq 3\sqrt[5]{6}$


Treasure every moment that you have!
And remember that Time waits for no one.
Yesterday is history. Tomorrow is a mystery.
Today is a gift. That’s why it’s called the present.


#134
Khoa Linh

Khoa Linh

    Thiếu úy

  • Thành viên
  • 601 Bài viết

Bài 49(Phạm Kim Hùng): Cho a, b, c dương và $a^2+b^2+c^2=3$. Chứng minh rằng:

$\frac{a}{a^2+2b+3}+\frac{b}{b^2+2c+3}+\frac{c}{c^2+2a+3}\leq \frac{1}{2}$

Bài 50(Phạm Kim Hùng): Cho 4 số dương a, b, c, d có tích bằng 1. Chứng minh:

$(a^2+1)(b^2+1)(c^2+1)(d^2+1)\geq (a+b+c+d)^2$

Mình xin đưa ra lời giải hai bài này.

Bài 49:

$\sum \frac{a}{a^2+2b+3}\leq \frac{1}{2}\sum \frac{a}{a+b+1}$

Bất đẳng thức tương tương: $\sum \frac{a}{a+b+1}\leq 1\Leftrightarrow \sum \frac{b+1}{a+b+1}\geq 2$

Áp dụng Cauchy - Schwarz ta có:

$\sum \frac{b+1}{a+b+1}\geq \frac{(a+b+c+3)^2}{(b+1)(a+b+1)+(c+1)(b+c+1)+(a+1)(c+a+1)}=1$ (khai triển tất cả kết hợp giả thiết $a^2+b^2+c^2=3$)

Bài 50:

Theo nguyên lý Dirichlet thì trong 4 số $a-1;b-1;c-1;d-1$ thì có hai số mà tích của chúng không âm. Giả sử $(b-1)(c-1)\geq 0$

Áp dụng Cauchy - Schwarz ta có:

$(a^2+1)(b^2+1)(c^2+1)(d^2+1)=(a^2c^2+a^2+c^2+1)(b^2d^2+1+b^2+d^2)\geq (abcd+a+bc+d)^2=(1+a+d+bc)^2$

Bất đẳng thức quy về chứng minh: $1+a+d+bc\geq a+b+c+d\Leftrightarrow (b-1)(c-1)\geq 0$ (đúng)


Bài viết đã được chỉnh sửa nội dung bởi Khoa Linh: 21-04-2018 - 00:24

$\sqrt[LOVE]{MATH}$

"If I feel unhappy, I do mathematics to become happy. If I am happy, I

 

do mathematics to keep happy" - Alfréd nyi 


#135
Khoa Linh

Khoa Linh

    Thiếu úy

  • Thành viên
  • 601 Bài viết

Bài 61: Cho các số thực x, y, z thỏa mãn $x^2+y^2+z^2=2$.

Chứng minh rằng: $x+y+z\leq xyz+2$

(IMO  Shortlist 1987)


Bài viết đã được chỉnh sửa nội dung bởi Khoa Linh: 21-04-2018 - 00:28

$\sqrt[LOVE]{MATH}$

"If I feel unhappy, I do mathematics to become happy. If I am happy, I

 

do mathematics to keep happy" - Alfréd nyi 


#136
tr2512

tr2512

    Thượng sĩ

  • Thành viên
  • 272 Bài viết

Bài 62

Cho các số thực $a, b, c$ bất kì. Chứng minh:

${\left( {{a^2} + {b^2} + {c^2} - ab - bc - ca} \right)^3} \ge {\left( {a - b} \right)^3}{\left( {b - c} \right)^3} + {\left( {b - c} \right)^3}{\left( {c - a} \right)^3} + {\left( {c - a} \right)^3}{\left( {a - b} \right)^3}$ (Trần Hoàng Nam)

@hoangkimca2k2: bạn xem lại bài 60 cái, cho a=b=c=1 thì VT<VP luôn rồi :D


Bài viết đã được chỉnh sửa nội dung bởi tr2512: 21-04-2018 - 07:20


#137
hoangkimca2k2

hoangkimca2k2

    Sĩ quan

  • Thành viên
  • 477 Bài viết

mk xin góp 1 bài 

Bài 59 : Cho a,b,c >0. Tìm GTNN của biểu thức

  P$\frac{a^3}{a^2+b^2}+\frac{b^3}{b^2+c^2}+\frac{c^3}{c^2+b^2}-\sqrt{a+b+c}$

 

Cách 2: $By AM-GM$

Cần CM: $\frac{a^3}{a^2+b^2}+\frac{b^3}{b^2+c^2}+\frac{c^3}{c^2+b^2}\geq \frac{a+b+c}{2}$

Ta có: $\sum \frac{a^{3}}{b^{2}+c^{2}}=\sum \left ( a-\frac{ab^{2}}{a^{2}+b^{2}} \right )\geq \frac{a+b+c}{2}$. tương tự rồi lm giống cách 1


Bài viết đã được chỉnh sửa nội dung bởi hoangkimca2k2: 21-04-2018 - 11:20

  N.D.P 

#138
tr2512

tr2512

    Thượng sĩ

  • Thành viên
  • 272 Bài viết

Bài 61: Cho các số thực x, y, z thỏa mãn $x^2+y^2+z^2=2$.

Chứng minh rằng: $x+y+z\leq xyz+2$

(IMO  Shortlist 1987)

Ta có: $2 = {a^2} + {b^2} + {c^2} \ge {a^2} + {b^2} \ge 2{\rm{a}}b \Rightarrow ab \le 1$

Áp dụng bất đẳng thức C-S có:

$a + b + c - abc = c\left( {1 - ab} \right) + a + b \le \sqrt {\left[ {{{\left( {ab - 1} \right)}^2} + 1} \right]\left[ {{{\left( {a + b} \right)}^2} + {c^2}} \right]} = \sqrt {\left( {{a^2}{b^2} - 2{\rm{a}}b + 2} \right)\left( {{c^2} + {a^2} + {b^2} + 2{\rm{a}}b} \right)} \\ = \sqrt {\left( {{a^2}{b^2} - 2{\rm{a}}b + 2} \right)\left( {2 + 2{\rm{a}}b} \right)} = \sqrt {2{a^3}{b^3} - 4{{\rm{a}}^2}{b^2} + 4{\rm{a}}b + 2{{\rm{a}}^2}{b^2} - 4{\rm{a}}b + 4} \\ = \sqrt {4 + 2{{\rm{a}}^3}{b^3} - 2{{\rm{a}}^2}{b^2}} = \sqrt {4 + 2{{\rm{a}}^2}{b^2}\left( {ab - 1} \right)} \le \sqrt 4 = 2$



#139
tr2512

tr2512

    Thượng sĩ

  • Thành viên
  • 272 Bài viết

Bài 63:

Cho $a,b,c$ là các số thực không âm thỏa mãn $a+b+c=1$. Chứng minh:

$a\sqrt {a + 2b} + b\sqrt {b + 2c} + c\sqrt {c + 2{\rm{a}}} \le 1$ Dấu bằng xảy ra khi nào ?

P/s: nhớ chỉ ra kĩ dấu bằng :D 



#140
tr2512

tr2512

    Thượng sĩ

  • Thành viên
  • 272 Bài viết

Bài 54Cho các số dương $a\,,\,b\,,\,c$. Chứng minh rằng: $\frac{a}{b}\,+ \,\frac{b}{c}\,+\, \frac{c}{a}\,+\, 1\,\geqq \,\frac{2\sqrt{2}}{3}\,(\sqrt{\frac{b+c}{a}}\,+\, \sqrt{\frac{c+a}{b}}\,+\, \sqrt{\frac{a+ b}{c}})$

Đặt: $x = \frac{a}{b};\,\,b = \frac{y}{z};\,\,c = \frac{z}{x} \Rightarrow xyz = 1$

Viết lại bất đẳng thức thành:

$x + y + z + 1 \ge \frac{{2\sqrt 2 }}{3}\left( {\sqrt {xy + z} + \sqrt {x{\rm{z}} + y} + \sqrt {yz + x} } \right)$

Áp dụng bất đẳng thức C-S:

$\sqrt {xy + z} + \sqrt {x{\rm{z}} + y} + \sqrt {yz + x} \le \sqrt {3\left( {x + y + z + xy + yz + z{\rm{x}}} \right)}$

Như vậy ta cần chứng minh:

$x + y + z + 1 \ge \frac{{2\sqrt 2 }}{3}\sqrt {3\left( {x + y + z + xy + yz + z{\rm{x}}} \right)} \\ \Leftrightarrow {\left( {x + y + z} \right)^2} + 2\left( {x + y + z} \right) + 1 \ge \frac{8}{3}\left( {x + y + z + x{\rm{z}} + yz + xy} \right)\\ \Leftrightarrow {x^2} + {y^2} + {z^2} + 1 \ge \frac{2}{3}\left( {x + y + z + xy + yz + z{\rm{x}}} \right)\\ \Leftrightarrow \frac{{{x^2} + 1}}{3} + \frac{{{y^2} + 1}}{3} + \frac{{{z^2} + 1}}{3} + \frac{{{x^2} + {y^2}}}{3} + \frac{{{y^2} + {z^2}}}{3} + \frac{{{z^2} + {x^2}}}{3} \ge \frac{2}{3}\left( {x + y + z + xy + yz + z{\rm{x}}} \right)$

Áp dụng bất đẳng thức AM-GM cho vế trái thu được ngay điều phải chứng minh.







Được gắn nhãn với một hoặc nhiều trong số những từ khóa sau: bất đẳng thức, holder, cosi, bunhiacopxki

1 người đang xem chủ đề

0 thành viên, 1 khách, 0 thành viên ẩn danh